Property Multiple Choice

Pataasin ang iyong marka sa homework at exams ngayon gamit ang Quizwiz!

A buyer entered into a valid written contract to purchase Blackacre, a large tract of land, from a seller for its fair market value of $50,000. The contract was assignable by the buyer. The buyer duly notified the seller to convey title to the buyer and her friend, whom the buyer had not seen for many years. When the buyer learned that her friend would have to sign certain documents in connection with the closing, she prevailed upon her brother to attend the closing and pretend to be her friend. The buyer and her brother attended the closing, and the seller executed an instrument in the proper form of a deed, purporting to convey Blackacre to the buyer and her friend, as tenants in common. The brother pretended that he was the friend, and he signed the friend's name to all the required documents. The buyer provided the entire $50,000 consideration for the transaction. The deed was promptly and properly recorded. Unknown to the buyer or her brother, the friend had died several months before the closing. The friend's will, which was duly probated, devised "All my real estate to my nephew" and the residue of his estate to the buyer. The buyer and the nephew have been unable to agree as to the status or disposition of Blackacre. The nephew brought an appropriate action against the seller and the buyer to quiet legal title to an undivided one-half interest in Blackacre. The court should hold that legal title to Blackacre is vested all in the seller. all in the buyer. one-half in buyer and one-half in the seller. one-half in the buyer and one-half in the nephew.

Answer choice C is correct. The seller attempted to convey Blackacre to the buyer and her friend in equal one half shares. Because the conveyance to the friend was ineffective due to the friend's death, the seller did not ever successfully convey title of that one half share. Thus, answer choices B and D are incorrect. Answer choice A is incorrect because the conveyance of the other half to the buyer was successful, so the two hold legal title to Blackacre equally.

The owner of Blueacre, a valuable tract of land located in York County, executed a document in the form of a warranty deed on Blueacre. The deed was regular in all respects except that the only language designating the grantees in each of the granting and habendum clauses was: "The leaders of all the Protestant Churches in York County." The instrument was acknowledged as required by statute and promptly and properly recorded. The owner told his lawyer, but no one else, that he had made the conveyance as he did because he abhorred sectarianism in the Protestant movement and because he thought that the leaders would devote the asset to lessening sectarianism. A week later, the owner died suddenly and unexpectedly, leaving a will that bequeathed and devised his entire estate to a friend. After probate of the will became final and the administration on the owner's estate was closed, the friend instituted an appropriate action to quiet title to Blueacre. He properly served each Protestant church situated in the county as defendant. The only evidence introduced consisted of: i) the chain of title under which the owner held ownership; ii) the probated will; iii) the recorded deed; iv) the fact that only the owner and his lawyer knew about the deed; and v) the conversation, described above, the owner had with his lawyer. In such action, judgment should be for the friend, because there is inadequate identification of grantees in the deed. the friend, because the state of the evidence would support a finding of delivery of the deed. the defendants, because a deed is prima facie valid until rebutted. the defendants, because recording established delivery prima facie until rebutted.

Answer choice A is correct. A deed that does not sufficiently identify the grantee is ineffective. Here, no name, position, and or even specific churches are identified with any certainty, so the deed is ineffective and Blueacre becomes part of the estate that passes to the friend. Answer choices B and D are incorrect because it was the identification of the grantees, not the delivery or recording of the deed that made the deed ineffective. Answer choice C is incorrect because substantial evidence exists to rebut the validity of the deed.

An investor purchased a tract of land, financing a large part of the purchase price by a loan from a business partner that was secured by a mortgage. The investor made the installment payments on the mortgage regularly for several years. Then the investor persuaded a neighbor to buy the land, subject to the mortgage to his partner. They expressly agreed that the neighbor would not assume, but agreed to pay the investor's debt to the partner. The investor's mortgage to the partner contained a due-on-sale clause stating, "If Mortgagor transfers his/her interest without the written consent of Mortgagee first obtained, then at Mortgagee's option the entire principal balance of the debt secured by this Mortgage shall become immediately due and payable." However, without seeking his partner's consent, the investor conveyed the land to the neighbor, the deed stating in pertinent part "..., subject to a mortgage to Partner," and giving details and recording data related to the mortgage. The neighbor took possession of the land and made several mortgage payments, which the partner accepted. Now, however, neither the neighbor nor the investor has made the last three mortgage payments. The partner has sued the neighbor for the amount of the delinquent payments. In this action, for whom should the court render judgment? The neighbor, because she did not assume and agree to pay the investor's mortgage debt. The neighbor, because she is not in privity of estate with the partner. The partner, because the investor's deed to the neighbor violated the due-on-sale clause. The partner, because the neighbor is in privity of estate with the partner.

Answer choice A is correct. A grantee that does not assume the mortgage, but rather takes subject to the mortgage, is not personally liable for the debt. In this case, there was no express assumption. In fact, the parties agreed that the neighbor was not assuming the mortgage debt. The investor is primarily liable as well as a surety for the debt. Answer choice D is incorrect because privity of estate arises when the parties share a relationship with the land (e.g., a landlord and a tenant), which is not present in these facts. However, the lack of privity between the neighbor and the partner has no effect on the mortgage debt. Thus, answer choice B is incorrect. Answer choice C is incorrect because a due-on-sale clause is used to accelerate the debt in the event of a transfer of the property to a third party. Here, the partner is suing only for delinquent payments.

A widower owns in fee simple a ranch, Ranchacre. The widower has one child, a son, who is married. The child has one child, the widower's grandchild, who is also married but has no children. In an effort to dispose of Ranchacre to his descendants and to honor a request by his grandchild that she be skipped in any disposition, the widower conveys Ranchacre to his son for life with the remainder to the grandchild's children in fee simple. What interest, if any, is created in favor of the grandchild's unborn children at the time of the conveyance? A contingent remainder. A vested remainder subject to divestment. A springing use. None.

Answer choice A is correct. A remainder is contingent if it is created in a grantee that is unascertainable, or if it is subject to an express condition precedent to grantee's taking. The grandchild does not have any children at the time of the conveyance, therefore, they are unascertainable and the remainder is contingent. Answer choice B is incorrect because a vested remainder requires an ascertainable grantee. Answer choice C is incorrect because a springing executory interest divests the interest of another or fills a gap in possession in which the estate reverts to grantor. In this case, the grandchild's children would not divest the widower of the property; they would merely take upon his death. Answer choice D is incorrect. This conveyance does not violate the Rule Against Perpetuities because the grandchild is the measuring life and the interest will vest, if at all, within 21 years of her death

A landlord leased an apartment to a tenant by written lease for two years ending on the last day of a recent month. The lease provided for $700 monthly rental. The tenant occupied the apartment and paid the rent for the first 15 months of the lease term, until he moved to a new job in another city. Without consulting the landlord, the tenant moved a friend into the apartment and signed an informal writing transferring to the friend his "lease rights" for the remaining nine months of the lease. The friend made the next four monthly $700 rental payments to the landlord. For the final five months of the lease term, no rent was paid by anyone, and the friend moved out with three months left on the lease term. The landlord was on an extended trip abroad, and did not learn of the default and the vacancy until last week. The landlord sued the tenant and the friend, jointly and severally, for $3,500 for the last five months' rent. What is the likely outcome of the lawsuit? Both the tenant and the friend are liable for the full $3,500, because the tenant is liable on privity of contract and the friend is liable on privity of estate as assignee. The friend is liable for $1,400 on privity of estate, which lasted only until he vacated, and the tenant is liable for $2,100 on privity of contract and estate for the period after the friend vacated. The friend is liable for $3,500 on privity of estate and the tenant is not liable, because the landlord's failure to object to the friend's payment of rent relieved the tenant of liability. The tenant is liable for $3,500 on privity of contract and the friend is not liable, because a sublessee does not have personal liability to the original landlord.

Answer choice A is correct. An assignment arises when a tenant transfers all or some of the leased premises to another for the remainder of the lease term, retaining no interest in the assigned premises. Any transfer of less than the duration of the lease is a sublease. Privity of contract is created by the signing of a lease, while the privity of estate relationship is connected through possession of the property. In this case, the tenant was in privity of contract (upon signing the lease) and privity of estate (upon taking possession) with the landlord. When the tenant transferred the remainder of his lease term to his friend, the friend was then in privity of estate with the landlord as to all covenants that run with the land, including the covenant to pay rent. Since the tenant continued to be in privity of contract because he was not released from his lease by the landlord, the tenant continued to remain fully liable to the landlord for the missing rent payments. The friend was liable for the defaulted rental payments as an assignee. Answer choice B is incorrect because during an assignment, both original tenant and assignee remain fully liable to the landlord. Answer choice C is incorrect because the landlord never released the tenant from the original lease. Answer choice D is incorrect because it assumes that the friend was a sublessee, which he was not. Here, as noted previously, the tenant transferred all the remaining time of the lease to the friend and retained no other interest.

Three brothers inherited, as joint tenants with the right of survivorship, a building in which their parents had operated a hardware store. Only the oldest brother continued to operate the hardware store on the premises, but he did not restrict his brothers' access to the building. The middle brother sold his interest in the building to the oldest brother. The youngest brother died, leaving everything to his daughter in his will. Who owns the building? A. The oldest brother in fee simple. B. The oldest brother and the youngest brother's daughter as joint tenants. C. The oldest brother and the youngest brother's daughter as equal tenants in common. D. The oldest brother and the youngest brother's daughter as tenants in common, with the oldest brother owning a 2/3 interest and the youngest brother's daughter owning a 1/3 interest.

Answer choice A is correct. Each brother inherited a 1/3 interest in the building. The middle brother's sale of his interest to the oldest brother severed the joint tenancy with respect to that interest. As a consequence, the oldest brother held a 1/3 interest as a tenant in common with the youngest brother. The oldest and youngest brothers held the remaining 2/3 interest as joint tenants. Upon the youngest brother's death, his interest passed automatically to the oldest brother by virtue of the right of survivorship. This interest, combined with the remaining 1/3 interest that oldest brother possessed, gave the older brother complete ownership of the building. Answer choices B, C, and D are incorrect because the youngest brother's joint tenancy interest in the building passed automatically to the oldest brother due to the right of survivorship. Consequently, the youngest brother did not have an interest in the building that he could devise to his daughter.

A man owned a large tract of land. Upon his death, the man devised a portion of the land to his daughter and the remaining portion to his son. The sister's tract contained a fishing pond. She told her brother that he could continue to use an existing path across her land to use the pond. Both siblings used the path and fished in the pond regularly. Ten years later, the sister decided to move out of state, and sold her land to her brother. Shortly thereafter, the brother sold the land that had been his sister's to a farmer, and then the other tract of land to a fisherman. Before the fisherman agreed to buy that tract of land, the brother told the fisherman that there was a pond on the farmer's land, and that sister had always let him access pond via the path. Neither deed mentioned an easement of any kind. The fisherman now claims that he has an easement to use the path and access the fishing pond. Does he? A. No, because the estates merged when the brother bought the sister's land. B. No, because the two tracts were originally part of the same large tract of land owned by the siblings' father. C. Yes, because the fisherman bought the tract only after the brother told him about the easement. D. Yes, because use of the path is necessary to access the fishing pond.

Answer choice A is correct. Easements are terminated when the servient estate merges with the easement, i.e., when the same owner owns both the easement and the servient estate. The easement is not revived when the ownership is subsequently split. Because the brother owned both the easement and the servient estate, and because the easement was never expressly recreated, the fisherman does not have an easement on the other owner's property. Answer choice B is incorrect because it is irrelevant that both tracts were owned by a sole owner before the easement was created. Merger is only relevant after the easement is created. Answer choice C is incorrect because the brother only told the fisherman about the easement that he had had. Since the brother did not own the land with the pond at the time of this conversation, the brother lacked the power to grant the easement to access the pond to the fisherman. Answer choice D is incorrect because this is not an easement by necessity. An easement by necessity arises when a tract of land is landlocked, and crossing the servient estate is the only way the owner of the dominant estate can access his own property. Nothing in the facts indicates that this is the case. The fact that the fisherman can only access the pond to which he has no right by crossing the farmer's property does not create an easement by necessity.

A testator owned Hilltop in fee simple. By his will, he devised as follows: "Hilltop to such of my grandchildren who shall reach the age of 21; and by this provision I intend to include all grandchildren whenever born." At the time of his death, the testator had three children and two grandchildren. Courts hold such a devise valid under the common-law Rule Against Perpetuities. What is the best explanation of that determination? A. All of the testator's children would be measuring lives. B. The rule of convenience closes the class of beneficiaries when any grandchild reaches the age of 21. C. There is a presumption that the testator intended to include only those grandchildren born prior to his death. D. There is a subsidiary rule of construction that dispositive instruments are to be interpreted so as to uphold interests rather than to invalidate them under the Rule Against Perpetuities.

Answer choice A is correct. Even if a grandchild was born more than 21 years after the testator's death, the interest would still vest or fail within 21 years of a child's life. Answer choice B is incorrect because the rule of convenience does not apply when the class is expressly defined to include individuals "whenever born;" the rule of convenience is only a rule of construction. Answer choice C is incorrect because the facts clearly state that the testator meant to include "all grandchildren whenever born." Answer choice D is incorrect because, although it is a true statement, answer choice A more specifically explains the reason why the interests do not violate the Rule Against Perpetuities.

A grantor owned two tracts of land, one of 15 acres and another of five acres. The two tracts were a mile apart. Fifteen years ago, the grantor conveyed the smaller tract to a grantee. The grantor retained the larger tract. The deed to the grantee contained, in addition to proper legal descriptions of both properties and identifications of the parties, the following: I, the grantor, bind myself and my heirs and assigns that in the event that the larger tract that I now retain is ever offered for sale, I will notify the grantee and his heirs and assigns in writing, and the grantee and his heirs and assigns shall have the right to purchase the larger tract for its fair market value as determined by a board consisting of three qualified expert independent real estate appraisers. With appropriate references to the other property and the parties, there followed a reciprocal provision that conferred upon the grantor and her heirs and assigns a similar right to purchase the smaller tract, purportedly binding the grantee and his heirs and assigns. Ten years ago, a corporation acquired the larger tract from the grantor. At that time, the grantee had no interest in acquiring the larger tract and by an appropriate written document released any interest he or his heirs or assigns might have had in the larger tract. Last year, the grantee died. The smaller tract passed by the grantee's will to his daughter. She has decided to sell the smaller tract. However, because she believes the corporation has been a very poor steward of the larger tract, she refuses to sell the smaller tract to the corporation even though she has offered it for sale in the local real estate market. The corporation brought an appropriate action for specific performance after taking all of the necessary preliminary steps in its effort to exercise its rights to purchase the smaller tract. The daughter asserted all possible defenses. The common law Rule Against Perpetuities is unmodified in the jurisdiction. If the court rules for the daughter, what is the reason? A. The provision setting out the right to purchase violates the Rule Against Perpetuities. B. The grantee's release 10 years ago operates as a waiver regarding any right to purchase that the corporation might have. C. The two tracts of land were not adjacent parcels of real estate, and thus the right to purchase is in gross and is therefore unenforceable. D. Noncompliance with a right to purchase gives rise to a claim for money damages, but not for specific performance.

Answer choice A is correct. Fifteen years ago, each of the parties granted a reciprocal right of first refusal (or a preemptive right) to the other. A right of first refusal is a conditional option. It provides that if the owner ever decides to sell the property, the one holding the right of first refusal has the right to purchase it. In this case, however, the right violates the common law Rule Against Perpetuities. The right to purchase is triggered by the decision of one to sell his or her land. In this case, that decision might occur more than 21 years after a life in being at the time the right was granted. Thus, under the common law, the right of first refusal is struck ab initio. The question notes that the common law Rule Against Perpetuities is unmodified in this jurisdiction. Thus, there are no applicable statutory reforms to the rule, and because the question is written with the daughter winning, any statute which may exempt a commercial transaction is inapplicable. Answer choice B is incorrect because ten years ago, the grantee had the right to purchase the larger tract of land under the right of first refusal (or a preemptive right) given to the grantee. The grantee chose not to exercise that right. The fact that the grantee chose not to exercise the right of first refusal has no effect on whether the grantor can exercise the reciprocal right of first refusal regarding the grantee's land. Answer choices C and D are incorrect because they are misstatements of law.

An owner had title to Greenview in fee simple. Without the owner's knowledge, a neighbor entered Greenview in 1980 and began to graze a flock of sheep he owned. After twelve years of possession of Greenview, the neighbor gave possession of Greenview to a buyer. At the same time, the neighbor also purported to transfer his sheep and all his interests in the land to the buyer by a document that was sufficient as a bill of sale to transfer personal property but was insufficient as a deed to transfer real property. One year later, the buyer entered into a lease with the owner to lease Greenview for a period of five years. After the end of the five-year term of the lease, the buyer remained on Greenview for an additional three years and then left Greenview. At that time, the owner conveyed Greenview by a quitclaim deed to a purchaser. The period of time to acquire title by adverse possession in the jurisdiction is ten years. After the owner's conveyance to the purchaser, title to Greenview was in the neighbor. the owner. the buyer. the purchaser.

Answer choice A is correct. For adverse possession to ripen into title, possession must be continuous, open and notorious, actual, exclusive, and hostile. The neighbor occupied Greenview for 12 years, grazed sheep on it, he did not share ownership with the owner, and his intent was to claim ownership as evidenced by the fact that he later attempted to give possession of Greenview to the buyer. Answer choice B is incorrect, because the use of the neighbor clearly indicates that adverse possession has taken place. Answer choice C is incorrect because the document the neighbor gave to the buyer was insufficient to transfer real property and the buyer did not satisfy the requirements of adverse possession on his own. Answer choice D is incorrect because the owner no longer has title to pass to the purchaser.

A landowner owned two adjoining parcels known as Lot 1 and Lot 2. Both parcels fronted on Main Street and abutted a public alley in the rear. Lot 1 was improved with a commercial building that covered all of the Main Street frontage of Lot 1; there was a large parking lot on the rear of Lot 1 with access from the alley only. Fifteen years ago, the owner leased Lot 1 to a tenant for 15 years. The tenant has continuously occupied Lot 1 since that time. Thirteen years ago, without the owner's permission, the tenant began to use a driveway on Lot 2 as a better access between Main Street and the parking lot than the alley. Eight years ago, the owner conveyed Lot 2 to a friend and, five years ago, the owner conveyed Lot 1 to the tenant by a deed that recited "together with all the appurtenances." Until last week, the tenant continuously used the driveway over Lot 2 to the tenant's parking lot in the rear of Lot 1. Last week the friend commenced construction of a building on Lot 2 and blocked the driveway used by the tenant. The tenant has commenced an action against the friend to restrain him from blocking the driveway from Main Street to the parking lot at the rear of Lot 1. The period of time to acquire rights by prescription in the jurisdiction is ten years. If the tenant loses, it will be because the owner owned both Lot 1 and Lot 2 until eight years ago. the tenant has access to the parking lot from the alley. mere use of an easement is not adverse possession. no easement was mentioned in the deed from the owner to the friend.

Answer choice A is correct. For the tenant to be successful with a claim of an easement by prescription, his use of the driveway would have to be open, continuous, and hostile for the entire 10-year statutory period. The tenant's use would likely be deemed permissive during the first landowner's ownership, because he did not object to the tenant's use of the driveway for years. Thus, the tenant's use could not become hostile until the friend's ownership. Because the friend purchased only eight years ago, the tenant's use does not meet the 10-year statutory prescription requirement, and the tenant would lose. Answer choice B is incorrect because access to the parking lot would only be relevant to a claim for easement by implication, not an easement by prescription. Answer choice C discusses adverse possession, which is not applicable to an easement by prescription claim (when exclusive use is not required). Answer choice D is incorrect because an easement by prescription is by definition not created expressly.

Three years ago, a seller conveyed Blackacre to a buyer for $50,000 by a deed that provided: "By accepting this deed, the buyer covenants for herself, her heirs and assigns, that the premises herein conveyed shall be used solely for residential purposes and, if the premises are used for nonresidential purposes, the seller, his heirs and assigns, shall have the right to repurchase the premises for the sum of one thousand dollars ($1,000)." In order to pay the $50,000 purchase price for Blackacre, the buyer obtained a $35,000 mortgage loan from the bank. The seller had full knowledge of the mortgage transaction. The deed and mortgage were promptly and properly recorded in proper sequence. The mortgage, however, made no reference to the quoted language in the deed. Two years ago, the buyer converted her use of Blackacre from residential to commercial without the knowledge or consent of the owner or of the bank. The buyer's commercial venture failed, and the buyer defaulted on her mortgage payments to the bank. Blackacre now has a fair market value of $25,000. The bank began appropriate foreclosure proceedings against the buyer. The seller properly intervened, tendered $1,000, and sought judgment that the buyer and the bank be ordered to convey Blackacre to the seller, free and clear of the mortgage. The common-law Rule Against Perpetuities is unmodified by statute. If the court rules against the seller, it will be because A. the provision quoted from the deed violates the Rule Against Perpetuities. B. the bank had no actual knowledge of, and did not consent to, the violation of the covenant. C. the rights reserved by the seller were subordinated, by necessary implication, to the rights of the bank as the lender of the purchase money. D. the consideration of $1,000 was inadequate.

Answer choice A is correct. It is possible that the seller's interest could vest outside the time frame required by the Rule Against Perpetuities (a life in being plus 21 years), because it may be hundreds of years before the premises are used for nonresidential purposes. Answer choice B is incorrect because consent is irrelevant to whether the covenant was violated. Answer choice C is incorrect because the seller's interest could not be subordinated by the bank. Answer choice D is incorrect because no consideration at all is required.

A father and son acquired as joint tenants a forty-acre parcel of land called Blackacre. They contributed equally to the purchase price. Several years later, the father proposed that they build an office building on Blackacre. The son rejected the proposal but orally agreed with the father that the father could go ahead on his own on the northerly half of Blackacre and the son could do what he wished with the southerly half of Blackacre. The father proceeded to build an office building on, and generally developed and improved, the northerly twenty acres of Blackacre. The son orally permitted the southerly twenty acres of Blackacre to be used by a local foundation as a bird sanctuary. The father died, leaving his entire estate to his daughter. The will named the son as executor of his will, but he refused to serve. In an appropriate action to determine the respective interests of the son and the daughter in Blackacre, if the son is adjudged to be the owner of all of Blackacre, the most likely reason for the judgment will be that the Statute of Frauds prevents the proof of the son's oral agreement. the father could not unilaterally sever the joint tenancy. the son's nomination as executor of the father's estate does not prevent him from asserting his claim against the daughter. the record title of the joint tenancy in Blackacre can be changed only by a duly recorded instrument.

Answer choice A is correct. Property can be partitioned voluntarily when the cotenants agree in writing on the division of land. Because the agreement was oral, the agreement cannot be proven. Answer choice B is incorrect because the father could unilaterally sever the joint tenancy either by a court action for partition or by conveyance during the father's lifetime to a third party. Answer choice C is incorrect because even if the property was properly partitioned, the son refused to serve as executor, and therefore has no fiduciary obligation to his sister. Answer choice D is incorrect because recording the agreement of division is not a requirement for a valid partition. The foregoing NCBE MBE question has been modified to reflect current NCBE stylistic approaches; the NCBE has not reviewed or endorsed this modification.

An owner had title to Brownacre in fee simple. Without the owner's knowledge, a neighbor entered Brownacre in 1950 and constructed an earthen dam across a watercourse. The earthen dam trapped water that the neighbor used to water a herd of cattle he owned. After twelve years of possession of Brownacre, the neighbor gave possession of Brownacre to a buyer. At the same time, the neighbor also purported to transfer his cattle and all his interests in the dam and water to the buyer by a document that was sufficient as a bill of sale to transfer personal property but was insufficient as a deed to transfer real property. One year later, the buyer entered into a lease with the owner to lease Brownacre for a period of five years. After the end of the five-year term of the lease, the buyer remained on Brownacre for an additional three years and then left Brownacre. At that time the owner conveyed Brownacre by a quitclaim deed to a purchaser. The period of time to acquire title by adverse possession in the jurisdiction is ten years. After the owner's conveyance to the purchaser, title to the earthen dam was in the person who then held title to Brownacre in fee simple. the buyer, as purchaser of the dam under the bill of sale. the person who then owned the water rights as an incident thereto. the neighbor, as the builder of the dam.

Answer choice A is correct. The dam is a fixture on the property, and structures built on the property and the building materials incorporated into that structure are part of the realty. A fixture that is essential to the use of the realty or that was specially designed for the realty is likely to be classified as a non-removable fixture. Answer choice B is incorrect because the bill of sale was not sufficient to transfer real property. Answer choice C is incorrect because regardless of the water rights, the dam is part of Brownacre. Answer choice D is incorrect because while the neighbor may be the owner of Brownacre by adverse possession, he does not own the dam merely because he was the builder of the dam.

A widow executes a will in which she leaves her house to her son and the remainder of her estate to her daughter. The house is subject to a purchase money mortgage, the unpaid portion of which is nearly equal to the value of the residuary estate. The son demands that the personal representative of the estate use the residuary estate to pay off the mortgage. The will contains a general provision for the payment of all the testator's debts, but not a specific provision authorizing the payment of the outstanding balance of the mortgage. Should the personal representative accede to the son's demand? A. Yes, because the son has a right to the exoneration of the mortgage. B. Yes, because the will contains a general provision for the payment of the testator's debts. C. No, because the doctrine of satisfaction does not apply to a specific devise. D. No, because the mortgage is a purchase-money mortgage.

Answer choice A is correct. The devisee of real property is entitled under the doctrine of exoneration of liens to have any outstanding balance of a mortgage or other encumbrance on the property to be paid from the remaining assets of the testator's estate. Answer choice B is incorrect because a general provision that requires the payment of a testator's debts does not require the personal representative of the estate to pay off any mortgages or other encumbrances on real property of the testator. Answer choice C is incorrect because the doctrine of satisfaction refers to the receipt of a gift from the testator prior to death that satisfies a devise made by the testator in the testator's will. Answer choice D is incorrect because the doctrine of exoneration of liens applies to all encumbrances, not just purchase-money mortgages.

A businessman executed a promissory note for $200,000 to a bank, secured by a mortgage on commercial real estate owned by the businessman. The promissory note stated that the businessman was not personally liable for the mortgage debt. One week later, a finance company obtained a judgment against the businessman for $50,000 and filed the judgment in the county where the real estate was located. At the time the judgment was filed, the finance company had no actual notice of the bank's mortgage. Two weeks after that filing, the bank recorded its mortgage on the businessman's real estate. The recording act of the jurisdiction provides: "Unless the same be recorded according to law, no conveyance or mortgage of real property shall be good against subsequent purchasers for value and without notice or against judgment creditors without notice." The finance company sued to enforce its judgment lien against the businessman's real estate. The bank intervened in the action, contending that the judgment lien was a second lien on the real estate and that its mortgage was a first lien. Is the bank's contention correct? A. No, because the judgment lien was recorded before the mortgage, and the finance company had no actual notice of the mortgage. B. No, because the businessman was not personally liable for the mortgage debt, and the mortgage was therefore void. C. Yes, because a mortgage prior in time has priority over a subsequent judgment lien. D. Yes, because the recording of a mortgage relates back to the date of execution of the mortgage note.

Answer choice A is correct. The judgment lien was recorded first in a jurisdiction that expressly protects judgment creditors without notice. The finance company had no actual notice of the mortgage and had no constructive notice because the mortgage was not recorded until two weeks after the judgment was filed. The bank's mortgage was not a purchase-money mortgage, which would have given it priority. Answer choice B is incorrect because the fact that the businessman was not personally liable for the mortgage debt is irrelevant and does not make the mortgage void. Answer choice C is incorrect because priority is determined under these facts by the order of filing. Answer choice D is incorrect because the recording of a mortgage does not relate back to the date of execution of the mortgage note. The mortgage gives constructive notice as of the date of its recording. Therefore, at the time the judgment was recorded, the finance company had neither actual nor constructive notice of the mortgage and is protected under the jurisdiction's recording act.

A landowner died, validly devising his land to his wife "for life or until remarriage, then to" their daughter. Shortly after the landowner's death, his daughter executed an instrument in the proper form of a deed, purporting to convey the land to her friend. A year later, the daughter died intestate, with her mother, the original landowner's wife, as her sole heir. The following month, the wife re-married. She then executed an instrument in the proper form of a deed, purporting to convey the land to her new husband as a wedding gift. Who now owns what interest in the land? The daughter's friend owns the fee simple. The wife owns the fee simple. The wife's new husband has a life estate in the land for the wife's life, with the remainder in the daughter's friend. The wife's new husband owns the fee simple.

Answer choice A is correct. The landowner's wife had a determinable life estate, evidenced by the words "for life" and "until remarriage" in the landowner's will. The daughter had a vested remainder and an executory interest. Both of the daughter's interests could be assigned to the friend. On the remarriage of the landowner's wife, the wife's life estate ended and it automatically went to the holder of the future interest, who was then the daughter's friend. Answer choice B is incorrect as a fee simple estate does not have words of special limitation. On the remarriage of the landowner's wife, the wife's life estate ended and it automatically went to the holder of the future interest. Answer choice C is incorrect because the wife remarried. Had the wife not remarried, the wife's life estate would have been transferable. Answer choice D is incorrect because the landowner's wife had no interest in the land to give to her new husband at the time she executed the deed.

An owner owned a tract of land called Blackacre. An old road ran through Blackacre from the abutting public highway. The road had been used to haul wood from Blackacre. Without the owner's permission and with no initial right, a neighbor, the owner of Whiteacre, which adjoined Blackacre, traveled over the old road for a period of 15 years to obtain access to Whiteacre, although Whiteacre abutted another public road. Occasionally, the neighbor made repairs to the old road. The period of time to acquire rights by prescription in the jurisdiction is ten years. After the expiration of 15 years, the owner conveyed a portion of Blackacre to a friend. The deed included the following clause: "together with the right to pass and repass at all times and for all purposes over the old road." The friend built a house fronting on the old road. The road was severely damaged by a spring flood, and the friend made substantial repairs to the road. The friend asked the neighbor and the owner to contribute one-third each to the cost of repairing the flood damage. They both refused, and the friend brought an appropriate action to compel contribution from the owner and the neighbor. In this action, the friend will lose as to both defendants. win as to both defendants. win as to the owner, but lose as to the neighbor. win as to the neighbor, but lose as to the owner.

Answer choice A is correct. The owner of an easement has a duty to maintain the easement for its purpose. The duty to contribute, however, is dependent upon the reasonableness of the repair. Specifically, the repairing party must give the contributing parties adequate notification and a reasonable opportunity to participate in decisions regarding the repairs. In this case, the neighbor and owner would not be required to contribute because there is no indication that they were given adequate notification or a reasonable opportunity to participate in decisions about the repair. Accordingly, answer choices B and D are incorrect. Answer choice C is incorrect because it is the owner of the easement, not the owner of the servient estate, who has the duty to maintain the easement, unless otherwise agreed. Here, there was an express easement with no mention of who would maintain the easement, so the responsibility rests with the friend.

A testator owned Blackacre, a vacant one-acre tract of land. Five years ago, he executed a deed conveying Blackacre to a church "for the purpose of erecting a church building thereon." Three years ago, the testator died leaving his son as his sole heir at law. His duly probated will left "all my Estate, both real and personal, to my friend." The friend was named. The church never constructed a church building on Blackacre and last month the church, for a valid consideration, conveyed Blackacre to a developer. The developer brought an appropriate action to quiet title against the son, the friend, and the church, and joined the appropriate state official. Such official asserted that a charitable trust was created which had not terminated. In such action, the court should find that title is now in the developer. the son. the friend. the state official.

Answer choice A is correct. The testator conveyed Blackacre to the church in fee simple, because the language "for the purpose of erecting a church building" is not clear or specific enough to create a fee simple determinable (which would revert to the testator's estate, and then the son or the friend when the church did not construct the building) or a charitable trust (which would be handled by the state official when the church tried to sell it). The church then conveyed Blackacre in fee simple to the developer, who now possesses title to Blackacre. Thus, answer choices B, C, and D are incorrect.

A seller and a buyer signed a contract for the sale of vacant land. The contract was silent concerning the quality of title, but the seller agreed in the contract to convey the land to the buyer by a warranty deed without any exceptions. When the buyer conducted a title search for the land, she learned that the applicable zoning did not allow for her planned commercial use. She also discovered that there was a recorded restrictive covenant limiting the use of the land to residential use. The buyer no longer wants to purchase the land. Must the buyer purchase the land? No, because the restrictive covenant renders the title unmarketable. No, because the zoning places a cloud on the title. Yes, because the buyer would receive a warranty deed without any exceptions. Yes, because the contract was silent regarding the quality of the title.

Answer choice A is correct. Unless the contract provides to the contrary, the law will imply that the seller will provide the buyer with a marketable title on the date of closing. A marketable title is not a perfect title but is a title a court will force an unwilling buyer to purchase. A right held in the land by a third party, such as the right to enforce a restrictive covenant, renders the title unmarketable, and the buyer need not purchase the land. Answer choice B is incorrect because, although in some cases an existing violation of a zoning code may render title unmarketable, the mere existence of a zoning code does not render the title unmarketable or place a cloud on the title. Answer choice C is incorrect because after a buyer accepts the deed, the doctrine of merger prevents the buyer from raising the issue of marketability of title, and the buyer's remedy regarding title issues, if any, will be based on the deed. Answer choice D is incorrect because this contract was silent on the quality of title and therefore a marketable title will be implied.

A widow transferred land that she held in fee simple to her only heir, her nephew, for his life, to her nephew's wife for the wife's life if she survived him, and then to any of the nephew's children who reached the age of 21. The widow later died intestate. Shortly after the widow's death, her nephew and his wife had their first and only child, a daughter. Five years ago, immediately prior to dying, the nephew's wife transferred her interest in the land to her daughter. One year ago, when the daughter was 18 years old and possessed legal capacity to transfer real property, she sold any interest she then owned in the land to a speculator. Recently, the nephew died and left everything to his daughter by his will. The jurisdiction recognizes the majority rule regarding inter vivos transfers of contingent remainders and executory interests. Which of the following would be the daughter's best argument that she is entitled to current possession of the land? A. The daughter was transferee of her mother's interest in the land. B. The daughter took the nephew's interest in the land upon his death. C. The daughter's contingent remainder in the land was not transferable inter vivos. D. The daughter was the only child of the nephew and his wife.

Answer choice B is correct. At the time of the widow's death, the nephew held a life estate, his wife a contingent life estate, any children of the nephew a contingent remainder, and the widow's estate a reversionary interest. Because the widow died intestate, the reversionary interest passed to the widow's only heir, the nephew. Upon the nephew's death, his life estate terminated and the reversionary interest was transformed in the current possessory interest in the land (i.e., a fee simple subject to an executory interest). This fee simple estate passed by the terms of the father's will to the daughter. However, at the time of the daughter's sale of any interests in the land to the speculator, the daughter had only a contingent remainder, which, upon her father's death, became a springing executory interest. This interest is not a current possessory interest. It will become one, if at all, only when the daughter reaches age 21. Accordingly, the speculator does not have a current possessory interest. Answer choice A is incorrect because the nephew's wife held only a contingent life estate in the land. This interest, which was transferred by the nephew's wife to her daughter inter vivos, terminated upon the death of the nephew's wife, since she did not survive her husband. Answer choice C is incorrect because, in most states, a contingent remainder may be transferred inter vivos. Answer choice D is incorrect because the daughter's status as a child of the nephew and his wife is relevant only with respect to her rights to the contingent remainder/springing executory interest, which is not a current possessory interest.

A man and woman were jointly in possession of Greenacre in fee simple as tenants in common. They joined in a mortgage of Greenacre to a bank. The man erected a fence along what he considered to be the true boundary between Greenacre and the adjoining property, owned by a neighbor. Shortly thereafter, the man had an argument with the woman and gave up his possession to Greenacre. The debt secured by the mortgage had not been paid. The neighbor surveyed his land and found that the fence erected a year earlier by the man did not follow the true boundary. Although part of the fence was within Greenacre, part of the fence encroached on the neighbor's land. The neighbor and the woman executed an agreement fixing the boundary line in accordance with the fence constructed by the man. The agreement, which met all the formalities required in the jurisdiction, was promptly and properly recorded. A year after the agreement was recorded, the man temporarily reconciled his differences with the woman and resumed joint possession of Greenacre. Thereafter, the man repudiated the boundary line agreement and brought an appropriate action against the neighbor and the woman to quiet title along the original true boundary. In such action, the man will win, because the bank was not a party to the agreement. win, because one tenant in common cannot bind another tenant in common to a boundary-line agreement. lose, because the agreement, as a matter of law, was mutually beneficial to the woman and the man. lose, because the woman was in sole possession of said premises at the time the agreement was signed.

Answer choice B is correct. Each cotenant must agree to bind the property to a boundary agreement. Neither the woman nor the neighbor can bind the man with the woman's signature, regardless of whether the bank was a party, or the extent of the benefit to the property. Answer choice A is incorrect because the bank was not a necessary party. Answer choice C is incorrect because it is a misstatement of the law. Answer choice D is incorrect because the woman's sole possession of Greenacre did nothing to diminish the man's interest.

Lawnacre was conveyed to two grantees by a deed that, in the jurisdiction in which Lawnacre is situated, created a co-tenancy in equal shares and with the right of survivorship. The jurisdiction has no statute directly applicable to any of the problems posed. One grantee, by deed, conveyed "my undivided one-half interest in Lawnacre" to her friend. That grantee has since died. In an appropriate action between the friend and the other grantee in which title to Lawnacre is at issue, the other grantee will prevail, because he is the sole owner of Lawnacre. prevail if, but only if, the co-tenancy created in the two grantees was a tenancy by the entirety. not prevail if he had knowledge of the conveyance prior to the grantee's death. not prevail, because the friend and the other grantee own Lawnacre as tenants in common.

Answer choice B is correct. Tenancy by the entirety is a joint tenancy between married persons with a right of survivorship. Because neither party in a tenancy by the entirety can alienate the property without the consent of the other, answer choice B provides the truest statement. Answer choice A is incorrect because even if the two grantees had held the property as a joint tenancy, the second grantee would then own the property with the friend as tenants in common. Answer choice C is incorrect because knowledge of a conveyance is not a requirement in severing a co-tenancy. Answer choice D would be correct if the two grantees had owned Lawnacre as a joint tenancy. However, answer choice D makes the unfounded assumption that the original co-tenancy was a joint tenancy, not allowing for the possibility that it might have been a different type of joint tenancy. Because answer choice B allows for the fact that a different outcome would result if this was a different type of co-tenancy, answer choice B is the better answer.

A seller contracted with a buyer, in a signed writing, to sell Greenacre, a 500-acre tract of farmland. The contract provided for exchange of the deed and for the purchase price of $500,000 in cash on January 15. Possession was to be given to the buyer on the same date. On January 15, the seller notified the buyer that because the tenant on Greenacre wrongfully refused to quit the premises until January 30, the seller would be unable to deliver possession of Greenacre until then, but he assured the buyer that he would tender the deed and possession on that date. When the seller tendered the deed and possession on January 30, the buyer refused to accept either, and refused to pay the $500,000. Throughout the month of January, the market value of Greenacre was $510,000, and its fair monthly rental value was $5,000. Will the seller probably succeed in an action against the buyer for specific performance? Yes, because the court will excuse the delay in tender on the ground that there was a temporary impossibility caused by the tenant's holding over. Yes, because time is ordinarily not of the essence in a land-sale contract. No, because the seller breached by failing to tender the deed and possession on January 15. No, because the seller's remedy at law for monetary relief is adequate.

Answer choice B is correct. The failure to close on a specified date is not considered breach, unless the parties expressly state that "time is of the essence." The seller is entitled to specific performance because the 15-day delay was reasonable, and time was not of the essence. Answer choice A is incorrect because no impossibility existed; the seller could have evicted the holdover tenant. Answer choice C is incorrect because the failure to close on a specified date is not considered a breach. Answer choice D is incorrect because specific performance is an available remedy to a seller in the event of breach by the buyer.

For valuable consideration the owner of Riveracre signed and gave to a friend a duly executed instrument that provided as follows: "The grantor may or may not sell Riveracre during her lifetime, but at her death, or if she earlier decides to sell, the property will be offered to Friend at $500 per acre. Friend shall exercise this right, if at all, within sixty days of receipt of said offer to sell." The friend recorded the instrument. The instrument was not valid as a will. Is the friend's right under the instrument valid? Yes, because the instrument is recorded. Yes, because the friend's right to purchase will vest or fail within the period prescribed by the Rule Against Perpetuities. No, because the friend's right to purchase is a restraint on the owner's power to make a testamentary disposition. No, because the friend's right to purchase is an unreasonable restraint on alienation.

Answer choice B is correct. The friend's interest does not violate the Rule Against perpetuities. Answer choice A is incorrect because it is a misstatement of law--recording an instrument does not alone make it valid. Answer choices C and D are both incorrect because the owner imposed the restraint on herself. Rules against restraint on alienation are to prevent imposing restraints on others.

An owner conveyed Blackacre to a woman by a warranty deed. The woman recorded the deed four days later. After the conveyance but prior to the woman's recording of the deed, a man properly filed a judgment against the original owner. The two pertinent statutes in the jurisdiction provide the following: 1) any judgment properly filed shall, for ten years from filing, be a lien on the real property then owned or subsequently acquired by any person against whom the judgment is rendered, and 2) no conveyance or mortgage of real property shall be good against subsequent purchasers for value and without notice unless the same be recorded according to law. The recording act has no provision for a grace period. The man joined both the original owner and the woman in an appropriate action to foreclose the judgment lien against Blackacre. If the man is unsuccessful, it will be because the original owner's warranty of title to the woman defeats the man's claim. the man is not a purchaser for value. any deed is superior to a judgment lien. four days is not an unreasonable delay in recording a deed.

Answer choice B is correct. The man's lien did not apply to the original owner because Blackacre was not owned by the original owner at the time the man filed the lien. If the man was a subsequent purchaser for value, the second statute would protect his claim by making the transfer of Blackacre to the woman ineffective for the purpose of the judgment lien. However, if the man was not a purchaser for value, his claim would be unsuccessful. Answer choice A is incorrect because even if the original owner breached the covenant against encumbrances within the warranty deed, the result would only entitle the woman to recover. The breach would have no effect on the man's claim. Answer choice C is a misstatement of law. Answer choice D is incorrect because the facts state that there is no grace period, so the reasonableness of the delay is irrelevant.

A brother and sister acquired as joint tenants a twenty-acre parcel of land called Greenacre. They contributed equally to the purchase price. Several years later, the brother proposed that they build an apartment development on Greenacre. The sister rejected the proposal but orally agreed with the brother that the brother could go ahead on his own on the northerly half of Greenacre and the sister could do what she wished with the southerly half of Greenacre. The brother proceeded to build an apartment development on, and generally developed and improved, the northerly ten acres of Greenacre. The sister orally permitted the southerly ten acres of Greenacre to be used by the Audubon Society as a nature preserve. The brother died, leaving his entire estate to his son. The will named the sister as executrix of his will, but she refused to serve. In an appropriate action to determine the respective interests of the sister and the son in Greenacre, if the son is adjudged to be the owner of the northerly ten acres of Greenacre, the most likely reason for the judgment will be that the close blood relationship between the sister and the brother removes the necessity to comply with the Statute of Frauds. the sister's conduct during the brother's lifetime estops her from asserting title to the northerly half of Greenacre. the joint tenancy was terminated by the oral agreement of the sister and the brother at the time it was made. the sister had a fiduciary obligation to her nephew by reason of her being named executrix of the brother's will.

Answer choice B is correct. The sister and brother did not properly partition their land, so the son's best argument is based in equity. Answer choice A is incorrect because property owned in joint tenancy can be partitioned voluntarily only when the parties agree in writing on the division of the land. A blood relationship has no effect on the Statute of Frauds. Answer choice C is incorrect because an oral agreement alone cannot sever a joint tenancy. Answer choice D is incorrect because even if the property was properly partitioned, the sister refused to serve as executrix, and therefore has no fiduciary obligation to her nephew.

At the time of his death last week, a testator owned Blackacre, a small farm. By his duly probated will executed five years ago, the testator did the following: 1. devised Blackacre to the testator's coworker for the life of the coworker's son, then to the testator's neighbor; 2. gave "all the rest, residue and remainder of my Estate, both real and personal" to a friend. At his death, the testator was survived by the coworker, the neighbor, the testator's son and sole heir, and the friend. The coworker's son had died a week before the testator. Title to Blackacre is now in the coworker for life, remainder to the neighbor. the neighbor, in fee simple. the testator's son, in fee simple. the friend, in fee simple.

Answer choice B is correct. The testator devised a life estate to the coworker for the life of the coworker's son's (pur autre vie). The coworker's life estate terminated upon his son's death, so he cannot take Blackacre, and the fee simple goes to the neighbor as the holder of the vested remainder. Thus, answer choice A is incorrect. The testator's son and the friend take nothing because the testator made a valid devise of Blackacre to the coworker, and then to the neighbor. Thus, answer choices C and D are incorrect because, although no anti-lapse statute was mentioned, the gift does not lapse even though the coworker's son predeceased the testator, as an alternative was specified in the will.

Under the terms of his duly probated will, a testator devised his house to his "grandchildren in fee simple" and the residue of his estate to his brother. The testator had had two children, a son and a daughter, but only the daughter survived the testator. At the time of the testator's death, the daughter was 30 years old and had two minor children (grandchildren of the testator) who also survived the testator. A third grandchild of the testator, who was the child of the testator's predeceased son, had been alive when the testator executed the will, but had predeceased the testator. Under the applicable intestate succession laws, the deceased grandchild's sole heir was his mother. A statute of the jurisdiction provides as follows: "If a devisee, including a devisee of a class gift, who is a grandparent or a lineal descendant of a grandparent of the testator is dead at the time of execution of the will or fails to survive the testator, the issue of such deceased devisee shall take the deceased's share under the will, unless the will expressly provides that this statute shall not apply. For this purpose, words of survivorship, such as 'if he survives me,' are a sufficient expression that the statute shall not apply." Who now owns the house? The testator's brother. The testator's two surviving grandchildren. The testator's two surviving grandchildren and all other grandchildren who are born to the testator's daughter. The testator's two surviving grandchildren and the deceased grandchild's mother.

Answer choice B is correct. The testator devised his house, which he owned at his death, to his grandchildren as a class gift. The testator was survived by two grandchildren who became the sole surviving members of the class. The deceased grandchild's interest lapsed because the grandchild, though alive at the time of the will execution, died before the testator. The deceased grandchild would qualify under the anti-lapse statute, but he was not survived by any issue who would be substituted for him under the statute. Answer choice A is incorrect because the house will pass to the two surviving grandchildren as surviving members of the class and will not fall into the residue. Answer choice C is incorrect because the class closed at the time of the testator's death and the two surviving members of the class will take. Additional children of the testator's daughter will not take. Answer choice D is incorrect. The anti-lapse statute would allow the deceased grandchild's share to pass to his issue, but not his mother.

The owner in fee simple of an undeveloped lot in the city promised to leave the property to her nephew upon her death. The nephew, shortly before his aunt's death, purported to convey the lot by a general warranty deed to his son as a gift. This transfer was not recorded. Despite the aunt's promise, the lot passed to a charity upon her death pursuant to provisions in her will. The following month, the son sold the lot to a developer but was shortly thereafter killed in an accident. When the charity's attorneys learned of the developer's plan to construct a building on the lot, they sued to block the construction by alleging the charity owned the lot. Which of the following breach of warranty actions can the developer successfully pursue against the nephew as a consequence of the charity's suit? A. Breach of the warranty against encumbrances. B. Breach of the warranty of quiet enjoyment. C. Breach of the warranty of the right to convey. D. Breach of the warranty of seisin.

Answer choice B is correct. The warranty of quiet enjoyment is a future covenant that is breached only upon interference with possession. It runs to successive grantees, such as the developer, as well as to the original grantee. Since the charity action interferes with the developer's possession of the lot, this warranty has been breached, and the developer may assert that breach against the nephew. Answer choice A is incorrect because the warranty against encumbrances is a present covenant that is breached by the existence of an encumbrance, such as an easement, at the time of conveyance of the property by the grantor. The charity's lawful ownership of the lot does not constitute an encumbrance, and in most states, a subsequent grantee (here, the developer) cannot sue the original grantor (here, the nephew) for breach of a present covenant. Answer choice C is incorrect because although the nephew did breach the warranty of the right to convey, this warranty is a present covenant. In most states, a subsequent grantee cannot sue the original grantor for breach of a present covenant. Answer choice D is incorrect because although the nephew did breach the warranty of seisin, the developer would not be able to sue the nephew for breach of a present covenant in most states.

A man obtained a bank loan secured by a mortgage on an office building that he owned. After several years, the man conveyed the office building to a woman, who took title subject to the mortgage. The deed to the woman was not recorded. The woman took immediate possession of the building and made the mortgage payments for several years. Subsequently, the woman stopped making payments on the mortgage loan, and the bank eventually commenced foreclosure proceedings in which the man and the woman were both named parties. At the foreclosure sale, a third party purchased the building for less than the outstanding balance on the mortgage loan. The bank then sought to collect the deficiency from the woman. Is the bank entitled to collect the deficiency from the woman? A. No, because the woman did not record the deed from the man. B. No, because the woman is not personally liable on the loan. C. Yes, because the woman took immediate possession of the building when she bought it from the man. D. Yes, because the woman was a party to the foreclosure proceeding.

Answer choice B is correct. The woman took title to the office building subject to the mortgage but did not assume the mortgage debt. The debt is to be satisfied out of the building. The building is the principal, and the man, as transferor, is the only party liable for any deficiency. This situation can be contrasted with one in which a buyer expressly assumes the mortgage debt. In that case, the buyer would be primarily liable for any deficiency and the seller, absent a release by the mortgagee, would be secondarily liable. Answer choice A is incorrect. The woman took title to the office building subject to the mortgage debt, which means that the debt was to be satisfied out of the building. Recording the deed would give the bank constructive notice of the transfer but would have no effect on the collection of the deficiency. Answer choice C is incorrect. The woman took title to the building subject to the mortgage. Her title to the building allowed her to take possession of the building, but her possession has no effect on the payment of any deficiency judgment. Taking title to the building subject to the mortgage means that the debt is to be satisfied out of the building. Answer choice D is incorrect. Because the woman took title to the building subject to the mortgage debt, she was a necessary party to the foreclosure proceeding. However, the fact that she took title to the building subject to the mortgage means that the debt is to be satisfied out of the building.

An owner owned Blackacre in fee simple, as the land records showed, when he contracted to sell Blackacre to a buyer. Two weeks later, the buyer paid the agreed price and received a warranty deed. A week thereafter, when neither the contract nor the deed had been recorded and while the owner remained in possession of Blackacre, a creditor properly filed her money judgment against the owner. She knew nothing of the buyer's interest. A statute in the jurisdiction provides: "Any judgment properly filed shall, for ten years from filing, be a lien on the real property then owned or subsequently acquired by any person against whom the judgment is rendered." The recording act of the jurisdiction provides: "No conveyance or mortgage of real property shall be good against subsequent purchasers for value and without notice unless the same be recorded according to law." The creditor brought an appropriate action to enforce her lien against Blackacre in the buyer's hands. If the court decides for the buyer, it will most probably be because the doctrine of equitable conversion applies. the jurisdiction's recording act does not protect creditors. the owner's possession gave the creditor constructive notice of the buyer's interest. the buyer was a purchaser without notice.

Answer choice B is correct. This is a notice jurisdiction. The buyer did not record before the creditor filed her judgment, so the buyer would lose unless there was an exception for creditors. If the recording statute protects the creditor, then the creditor would prevail. Answer choice A is incorrect because the doctrine of equitable conversion does not relate to priorities under the recording statute. Answer choice C doesn't make any sense, as the creditor would have no way of knowing of the buyer's interest simply by learning of the owner's possession. Answer choice D is incorrect because the question is whether the creditor had notice, not whether the buyer had notice.

An aunt, who was the owner in fee simple of Richacre, a large parcel of vacant land, executed a deed purporting to convey Richacre to her nephew. She told the nephew who was then 19, about the deed and said that she would give it to him when he reached 21 and had received his undergraduate college degree. Shortly afterward the nephew searched his aunt's desk, found and removed the deed, and recorded it. A month later, the nephew executed an instrument in the proper form of a warranty deed purporting to convey Richacre to his fiancée. He delivered the deed to his fiancée, pointing out that the deed recited that it was given in exchange for "$1 and other good and valuable consideration," and that to make it valid the fiancée must pay him $1. The fiancée, impressed and grateful, did so. Together, they went to the recording office and recorded the deed. The fiancée assumed the nephew had owned Richacre, and knew nothing about the nephew's dealing with his aunt. Neither the aunt's deed to the nephew nor the nephew's deed to the fiancée said anything about any conditions. The recording act of the jurisdiction provides: "No conveyance or mortgage of real property shall be good against subsequent purchasers for value and without notice unless the same be recorded according to law." Two years passed. The nephew turned 21, and then graduated from college. At the graduation party, the aunt was chatting with the fiancée and for the first time learned the foregoing facts. The age of majority in the jurisdiction is 18 years. The aunt brought an appropriate action against the fiancée to quiet title to Richacre. The court will decide for the aunt, because the nephew's deed to the fiancée before the nephew satisfied his aunt's conditions was void, as the fiancée had paid only nominal consideration. the aunt, because her deed to the nephew was not delivered. the fiancée, because the nephew has satisfied the aunt's oral conditions. the fiancée, because the deed to her was recorded.

Answer choice B is correct. Title did not pass to the nephew because there was no delivery of the deed (i.e., the aunt's use of conditional language made it clear that she did not have the intent to make a present transfer of title). Because the nephew could not have passed title to the fiancée, she could not hold title, and answer choices C and D are incorrect. Even if the nephew had title, the fiancée could not recover by estoppel because she was not a bona fide purchaser for value. Answer choice A is an incorrect statement of law because no consideration is necessary to transfer title.

The plaintiff owned Greenacre, a tract of land, in fee simple. The plaintiff executed an instrument in the proper form of a deed, purporting to convey Greenacre to the defendant in fee simple. The instrument recited that the conveyance was in consideration of "$5 cash in hand paid and for other good and valuable consideration." The plaintiff handed the instrument to the defendant and the defendant promptly and properly recorded it. Two months later, the plaintiff brought an appropriate action against the defendant to cancel the instrument and to quiet title. In support, the plaintiff proved that no money in fact had been paid by the defendant, notwithstanding the recitation, and that no other consideration of any kind had been supplied by the defendant. In such action, the plaintiff should lose, because any remedy the plaintiff might have had was lost when the instrument was recorded. lose, because the validity of conveyance of land does not depend upon consideration being paid, whether recited or not. prevail, because the recitation of consideration paid may be contradicted by parol evidence. prevail, because recordation does not make a void instrument effective.

Answer choice B is correct. Unlike a contract, consideration is not required for a deed to be valid, so the plaintiff's argument is moot. Answer choice C is incorrect for the same reason. Accordingly, there is no need to introduce parol evidence regarding consideration. Answer choices A and D are incorrect because recording a deed is also not required for that deed to be valid, and is not relevant to whether the transfer from the plaintiff to the defendant was valid.

The owner of Greenacre, a large tract of land, entered into a binding written contract with a buyer for the sale and purchase of Greenacre for $125,000. The contract required the seller to convey marketable record title. The buyer decided to protect his interest and promptly and properly recorded the contract. Thereafter, but before the date scheduled for the closing, an injured man obtained and properly filed a final judgment against the seller in the amount of $1 million in a personal injury suit. A statute in the jurisdiction provides: "Any judgment properly filed shall, for ten years from filing, be a lien on the real property then owned or subsequently acquired by any person against whom the judgment is rendered." The recording act of the jurisdiction authorizes recording of contracts and also provides: "No conveyance or mortgage of real property shall be good against subsequent purchasers for value and without notice unless the same be recorded according to law." There are no other relevant statutory provisions. At the closing, the buyer declined to accept the title of the seller on the ground that the injured man's judgment lien encumbered the title he would receive and rendered it unmarketable. The seller brought an appropriate action against the buyer for specific performance of the contract and joined the injured man as a party. In this action, the judgment should be for the seller, because in equity a purchaser takes free of judgment liens. the seller, because the contract had been recorded. the buyer, because the seller cannot benefit from the buyer's action in recording the contract. the buyer, because the statute creating judgment liens takes precedence over the recording act.

Answer choice B is correct. When the buyer recorded the contract, he put all subsequent purchasers, including the injured man, on notice of his equitable title. Because the injured man is not a bona fide purchaser, and thus not protected by the recording statute, he cannot burden the title. Answer choice A is incorrect because the fact that an equitable remedy is sought is not the reason that title to Greenacre is not burdened by the injured man's claim. Purchasers do not take free from all judgment liens. Answer choice C is an incorrect statement of law, as the issue would not be decided on this basis. Answer choice D incorrectly suggests that the statutes are inconsistent with each other such that one would take priority over the other. Instead, the recording statute merely states when a subsequent judgment lien attaches to property.

Two people owned Brownacre as joint tenants with the right of survivorship. The first joint tenant executed a mortgage on Brownacre to a lender in order to secure a loan. Subsequently, but before the indebtedness was paid to the lender, the first joint tenant died intestate with her daughter as her only heir at law. The jurisdiction at which Brownacre is located recognizes the title theory of mortgages. In an appropriate action, the court should determine that title to Brownacre is vested in the second tenant, with the entire interest subject to the mortgage. in the second tenant, free and clear of the mortgage. half in the second tenant, free of the mortgage and half in the daughter, subject to the mortgage. half in the second tenant and half in the daughter, with both subject to the mortgage.

Answer choice C is correct. A joint tenant may grant a mortgage interest in the joint tenancy property to a creditor. In the "title theory" states (the minority), mortgage severs the title and the tenancy between the joint tenants and creditor is converted into a tenancy in common. Consequently, the second tenant owns half, as a tenant in common, free of the mortgage. The daughter inherits the other half, along with the mortgage, because the joint tenancy was severed. Answer choice A is incorrect because the joint tenancy was severed with the mortgage. Answer choice B is incorrect because this is not a "lien theory" jurisdiction. In the "lien theory" states (the majority), the mortgage is only a lien on the property and does not sever the joint tenancy absent a default and foreclosure sale. Note that any lien against one joint tenant's interest also terminates upon that tenant's death, such that the lien does not encumber the surviving tenants' interest. Answer choice D is incorrect because the second tenant's half of the tenancy is not subject to the mortgage under the "lien theory" or "title theory."

Blackacre is a large tract of land owned by a religious order. On Blackacre, the religious order erected a large residential building where its members reside. Blackacre is surrounded by rural residential properties and its only access to a public way is afforded by an easement over a strip of land 30 feet wide. The easement was granted to the religious order by deed from the owner of one of the adjacent residential properties. The religious order built a driveway on the strip, and the easement was used for 20 years without incident or objection. Last year, as permitted by the applicable zoning ordinance, the religious order constructed a 200-bed nursing home and a parking lot on Blackacre, using all of Blackacre that was available for such development. The nursing home was very successful, and on Sundays visitors to the nursing home overflowed the parking facilities on Blackacre and parked all along the driveway from early in the morning through the evening hours. After two Sundays of the resulting congestion and inconvenience, the neighbor who granted the easement erected a barrier across the driveway on Sundays preventing any use of the driveway by anyone seeking access to Blackacre. The religious order objected. The neighbor brought an appropriate action to terminate the easement. The most likely result in this action is that the court will hold for the neighbor, because the religious order excessively expanded the use of the dominant tenement. the neighbor, because the parking on the driveway exceeded the scope of the easement. the religious order, because expanded use of the easement does not terminate the easement. the religious order, because the neighbor's use of self-help denies her the right to equitable relief.

Answer choice C is correct. An express easement can be terminated by release, merger, severance, abandonment, sale, destruction, prescription, and estoppel, but not by overuse. Although the new use of the driveway may be determined to exceed the scope of the easement (as suggested in answer choices A and B), the resulting remedy would not be termination of the easement, but rather an injunction more specifically restricting the use of the easement. Accordingly, both answer choices A and B are incorrect because exceeding the scope of the easement would not terminate the easement. Answer choice D is an incorrect statement of law.

A grantor conveyed her only parcel of land to a grantee by a duly executed and delivered warranty deed, which provided: "To have and to hold the described tract of land in fee simple, subject to the understanding that within one year from the date of the instrument said grantee shall construct and thereafter maintain and operate on said premises a public health center." The grantee constructed a public health center on the tract within the time specified and operated it for five years. At the end of this period, the grantee converted the structure into a senior citizens' recreational facility. It is conceded by all parties in interest that a senior citizens' recreational facility is not a public health center. In an appropriate action, the grantor seeks a declaration that the change in the use of the facility has caused the land and structure to revert to her. In this action, the grantor should win, because the language of the deed created a determinable fee, which leaves a possibility of reverter in the grantor. win, because the language of the deed created a fee subject to condition subsequent, which leaves a right of entry or power of termination in the grantor. lose, because the language of the deed created only a contractual obligation and did not provide for retention of property interest by the grantor. lose, because an equitable charge is enforceable only in equity.

Answer choice C is correct. Answer choice A is incorrect because the deed did not create a determinable fee. There was no durational language such as "so long as," "while," "during," or "until." Answer choice B is incorrect because the deed did not create a fee simple subject to a condition subsequent. There was no conditional language such as "provided that," "on condition that," or "but if." Thus, the deed did not retain any interest in the grantor. Answer choice D is incorrect because this is not an equitable charge.

A bank provided a loan to the purchaser of a parcel of undeveloped land. As security for the loan, the purchaser granted the bank a mortgage with respect to the parcel of land. Subsequently, the purchaser subdivided the land into three lots. The purchaser sold two of the three lots, each to a different buyer at a different time. Each buyer obtained a loan to finance the purchase from a different bank, neither of which was the bank that provided the original loan to the purchaser. Each buyer gave his lender a mortgage on his lot. The purchaser later defaulted on his bank loan. The bank filed an action to foreclose on all three parcels of land. The buyers of the two lots each filed an appropriate motion to protect their interests to the maximum extent possible. Which of the following best describes the likely outcome of the bank's attempt to proceed against the three lots? A. The bank may proceed against any of the three lots in any order that it chooses. B. The bank must proceed first against the lot retained by the purchaser and then against the lots in the order in which the mortgages on them were created. C. The bank must proceed first against the lot retained by the purchaser and then against the lots in the inverse order in which the mortgages on them were created. D. The bank must proceed against all of the lots in a manner that recovers an equal amount from each.

Answer choice C is correct. Generally, upon default of an obligation, a mortgagee may foreclose on any and all parcels of real property of the mortgagor that serve as security for the obligation. If there are junior security interests with respect to some of those parcels of real property, however, those junior interests may petition the court for protection of their interests under the doctrine of marshaling of assets. Under this doctrine, the holder of a senior security interest must first proceed against the property on which there are not any junior security interests, and then against the property on which the junior interest was more recently created, before proceeding against property on which the junior interest was more remotely created. Answer choice A is incorrect because, although a mortgagee may generally proceed with a foreclosure against any real property of the mortgagor that serves as security for the obligation, in this case the doctrine of marshalling of assets applies to prevent the mortgagee from exercising that discretion. Answer choice B is incorrect because the doctrine of marshalling of assets requires that the holder of the senior security interest proceed first against the property on which the junior interest was more recently created. Answer choice D is incorrect because the holder of the senior security interest must proceed first against property on which there are not any junior security interests, and then against the property on which the junior interest was more recently created, before proceeding against property on which the junior interest was more remotely created.

By a valid written contract, a seller agreed to sell land to a buyer. The contract stated, "The parties agree that closing will occur on next May 1 at 10 a.m." There was no other reference to closing. The contract was silent as to quality of title. On April 27, the seller notified the buyer that she had discovered that the land was subject to a longstanding easement in favor of a corporation for a towpath for a canal, should the corporation ever want to build a canal. The buyer thought it so unlikely that a canal would be built that the closing should occur notwithstanding this outstanding easement. Therefore, the buyer notified the seller on April 28 that he would expect to close on May 1. When the seller refused to close, the buyer sued for specific performance. Will the buyer prevail? No, because the easement renders the seller's title unmarketable. No, because rights of third parties are unresolved. Yes, because the decision to terminate the contract for title not being marketable belongs only to the buyer. Yes, because the seller did not give notice of the easement a reasonable time before the closing date.

Answer choice C is correct. If a contract of sale is silent as to quality of title, the court will imply a marketable title, and an easement does affect the marketability of title. But while the seller has a duty to deliver a marketable title, the requirement of marketable title is for the benefit of the buyer. The buyer may waive the right to have a marketable title, which is what the buyer did in this situation. Thus, answer choice C is correct and answer choice A is incorrect. Answer choice B is incorrect because it has no basis in law. Further, any risk would be on the buyer, and the buyer is the one who wants to proceed. Answer choice D is incorrect because the buyer waived the right to have a marketable title and is not asking the seller to remove the defect, so the buyer can enforce the contract.

A testator owned Maypond in fee simple. By her will, she devised as follows: "Maypond to such of my grandchildren who shall reach the age of 21; and by this provision I intend to include all grandchildren whenever born." At the time of her death, the testator had two children and five grandchildren. Which of the following additions to or changes in the facts of the preceding question would produce a violation of the common-law Rule Against Perpetuities? A posthumous child was born to the testator. The testator's will expressed the intention to include all afterborn grandchildren in the gift. The instrument was an inter vivos conveyance rather than a will. The testator had no grandchildren living at the time of her death.

Answer choice C is correct. If the instrument was an inter vivos conveyance, the perpetuities period will begin to run the date the instrument is created. After that date, the testator could have another child that would not be a life in being. Consequently, the testator could have a grandchild born more than 21 years after the death of a life in being. Answer choice A is incorrect because that child would still be a life in being. Answer choice B is incorrect because the facts already state that the testator meant to include "all grandchildren whenever born." Answer choice D is incorrect because the testator's children are the lives in being. Even if the testator never has grandchildren, the gift will fail within 21 years of a life in being. The foregoing NCBE MBE question has been modified to reflect current NCBE stylistic approaches; the NCBE has not reviewed or endorsed this modification.

A niece inherited vacant land from her uncle. She lived in a distant state and decided to sell the land to a colleague who was interested in purchasing the land as an investment. They orally agreed upon a price, and, at the colleague's insistence, the niece agreed to provide him with a warranty deed without any exceptions. The price was paid, the warranty deed was delivered, and the deed was promptly and properly recorded. Neither the niece nor the colleague had, at that point, ever seen the land. After recording the deed, the colleague visited the land for the first time and discovered that it had no access to any public right-of-way and that none of the surrounding lands had ever been held in common ownership with any previous owner of the tract of land. The colleague sued the niece for damages. For whom will the court find? The colleague, because lack of access makes title unmarketable. The colleague, because the covenants of warranty and quiet enjoyment in the deed were breached. The niece, because no title covenants were breached. The niece, because the agreement to sell was oral.

Answer choice C is correct. Lack of access may render title unmarketable under the contract of sale; however, the time to challenge marketable title is prior to the acceptance of the deed. Under the doctrine of merger, the remedy, if any, is on the title covenant in the deed. Lack of access does not violate any of the title covenants. The colleague received the title the niece said she had. No one had a superior title and thus the covenants of seisin, right to convey, quiet enjoyment, and general warranty were not breached. The covenant against encumbrances provides protection for interests held by third parties such as easements for access. The land was not subject to an express easement nor may any easement be implied based on either prior use or necessity because the lands were never held in common ownership. Answer choice A is incorrect as the doctrine of merger provides that once a deed has been accepted it is too late to sue on title matters under the contract of sale. Answer choice B is incorrect because the covenants of warranty and quiet enjoyment are breached when the grantee has been evicted from possession of the land by another with a superior title. The colleague has not been evicted by anyone with a superior title. Answer choice D correctly concludes that the niece will prevail but misstates the reasoning. The Statute of Frauds does require that an agreement to sell land be in writing. Nonetheless, if the parties have both fully performed under an oral contract, the relationship is the same as if the statute had been fully complied with initially. It is too late for the colleague, having accepted the deed, to now complain that the Statute of Frauds was not complied with because the agreement was oral.

Last year, a buyer and a seller entered into a valid contract for the sale of a parcel of real property. The contract contained no contingencies. The seller was killed in a car accident before the parcel was conveyed, but the closing eventually took place with the conveyance by a deed from the personal representative of the seller's estate. The personal representative of the seller's estate wants to distribute the proceeds of the real property sale. The seller's will was executed many years ago and was duly admitted to probate. Paragraph 5 of his will leaves all of the seller's real property to his son, and Paragraph 6 leaves the residue of the estate to the seller's daughter. No other provisions of the will are pertinent to the question regarding to whom the proceeds of the sale should be distributed. What will determine who receives the proceeds? Whether Paragraph 5 refers specifically to the parcel of real property that was sold or simply to "all of my real property." Whether the closing date originally specified in the contract was a date before or after the seller's death. Whether the jurisdiction has adopted the doctrine of equitable conversion. Whether the sale was completed in accordance with a court order.

Answer choice C is correct. Many events may occur during the executory time period, which is the time between the contract signing and the closing. If the seller dies during this time period leaving a will that devises the real estate to one person and the personalty to another, and if the contract contained no contingencies or all contingencies had been satisfied at the time of the death, the doctrine of equitable conversion applies. The determination of who will receive the proceeds thus depends on who had equitable title. In this case, the son had only a legal interest in the land at the time of the seller's death, and the daughter had the equitable interest in the proceeds and should receive them. Answer choice A is incorrect, because it does not matter whether Paragraph 5 refers specifically to the parcel sold or simply to "all of my real property." Answer choice B is incorrect, because it does not matter whether the closing date was scheduled before or after the seller's death. If the seller had died after the closing, the proceeds would have passed to the daughter, who received the residue of the estate. Answer choice D is incorrect because it does not matter whether the sale was completed in accordance with a court order or not.

An owner owned Goldacre, a tract of land, in fee simple. At a time when Goldacre was in the adverse possession of a hunter, a neighbor obtained the oral permission of the owner to use as a road or driveway a portion of Goldacre to reach adjoining land, Twin Pines, which the neighbor owned in fee simple. Thereafter, during all times relevant to this problem, the neighbor used this road across Goldacre regularly for ingress and egress between Twin Pines and a public highway. The hunter quit possession of Goldacre before acquiring title by adverse possession. Without any further communication between the owner and the neighbor, the neighbor continued to use the road for a total period, from the time he first began to use it, sufficient to acquire an easement by prescription. The owner then blocked the road and refused to permit its continued use. The neighbor brought suit to determine his right to continue use of the road. The neighbor should A. win, because his use was adverse to the hunter and once adverse it continued adverse until some affirmative showing of a change. B. win, because the neighbor made no attempt to renew permission after the hunter quit possession of Goldacre. C. lose, because his use was with permission. D. lose, because there is no evidence that he continued the adverse use for the required period after the hunter quit possession.

Answer choice C is correct. One of the required elements of adverse possession is hostility - that the adverse possessor possess the land with intent to claim ownership. The owner gave the neighbor permission to use the land, effectively negating the hostility requirement, and there is no evidence that the neighbor possessed the land with intent to claim ownership. Answer choices A and D are incorrect because the adverse possession must be against the true owner. Answer choice B is incorrect because the owner's permission to use the land did not change after the hunter quit possession.

An owner, who owned Blackacre in fee simple, conveyed Blackacre to a buyer by warranty deed. A neighbor, an adjoining owner, asserted title to Blackacre and brought an appropriate action against the buyer to quiet title to Blackacre. The buyer demanded that the owner defend the buyer's title under the deed's covenant of warranty, but the owner refused. The buyer then successfully defended at her own expense. The buyer brought an appropriate action against the owner to recover the buyer's expenses incurred in defending against the neighbor's action to quiet title to Blackacre. In this action, the court should decide for A. the buyer, because in effect it was the owner's title that was challenged. B. the buyer, because the owner's deed to her included the covenant of warranty. C. the owner, because the title the owner conveyed was not defective. D. the owner, because the neighbor may elect which of the owner or the buyer to sue.

Answer choice C is correct. The buyer can only recover from the owner if title were unmarketable. Here, it is clear that title was marketable because the buyer was successful in defending against the neighbor. The covenant of warranty only protects the buyer if it was breached by the owner due to defects in the title, which did not exist here. Thus, answer choices A and B are incorrect. Answer choice D is incorrect because the neighbor can only bring suit against the buyer as the owner of record.

An owner contracted to sell Vacantacre to a purchaser. The written contract required the owner to provide evidence of marketable title of record, specify a closing date, state that "time is of the essence," and provide that at closing, the owner would convey by warranty deed. The purchaser paid the owner $2,000 earnest money toward the $40,000 purchase price. The title evidence showed that an undivided one-eighth interest in Vacantacre was owned by another owner. The purchaser immediately objected to title and said he would not close on the owner's title. The owner responded, accurately, that the other owner was his daughter who would be trekking in Nepal until two weeks after the specified closing date. He said that she would gladly deed her interest upon her return, and that meanwhile his deed warranting title to all of Vacantacre would fully protect the purchaser. The owner duly tendered his deed but the purchaser refused to close. The purchaser brought an appropriate action to recover the $2,000 earnest money promptly after the specified closing date. The owner counterclaimed for specific performance, tendering a deed from himself and his daughter, who had by then returned. The court will hold for the owner, because his daughter's deed completing the transfer was given within a reasonable time. the owner, because his warranty deed would have given the purchaser adequate interim protection. purchaser, because the owner's title was not marketable and time was of the essence. the purchaser, because under the circumstances the earnest money amount was excessive.

Answer choice C is correct. The owner was obligated to deliver to the purchaser marketable title on the date of the closing. The owner's daughter still had interest in the land on the date of the closing, making title unmarketable. Answer choice A is incorrect because the "time is of the essence" clause in the agreement obligated the owner to deliver marketable title on the closing date, with no grace period for reasonableness. Answer choice B is incorrect because the warranty deed does not make the title marketable. Answer choice D is incorrect because the earnest money was not excessive, and because the owner failed to deliver marketable title.

Twenty years ago, a testator, who owned Blackacre, a one-acre tract of land, duly delivered a deed of Blackacre to a particular school district "so long as it is used for school purposes." The deed was promptly and properly recorded. Five years ago, the testator died leaving his son as his only heir but, by his duly probated will, he left "all my Estate to" a friend who was identified by name. Last month, the school district closed its school on Blackacre and for valid consideration duly executed and delivered a quitclaim deed of Blackacre to a new owner, who planned to use the land for commercial development. The new owner has now brought an appropriate action to quiet title against the son, the friend, and the school district. The only applicable statute is a provision in the jurisdiction's probate code which provides that any property interest which is descendible is devisable. In such action, the court should find that title is now held by the new owner. the son. the friend. the school district.

Answer choice C is correct. The testator conveyed to the school district a fee simple determinable, with a possibility of reverter in the testator (or his heirs and assigns) if Blackacre was ever not used for school purposes. Upon the closing of the school the condition was broken, and the property reverted to the testator (or his heirs). Thus, answer choice D is incorrect. The property reverted to the friend, not the son, per the testator's will, because the applicable statute indicates that the testator's property interest was devisable. Thus, answer choice B is incorrect. Answer choice A is incorrect because the property automatically reverted when the school closed, so the school district had no interest in the property when it gave the quitclaim deed to the new owner.

An owner owns his home, Blackacre, which was mortgaged to a bank by a duly recorded purchase money mortgage. Last year, the owner replaced all of Blackacre's old windows with new windows. Each new window consists of a window frame with three inserts: regular windows, storm windows, and screens. The windows are designed so that each insert can be easily inserted or removed from the window frame without tools to adjust to seasonal change and to facilitate the cleaning of the inserts. The new windows were expensive. The owner purchased them on credit, signed a financing statement, and granted a security interest in the windows to the supplier of the windows. The supplier promptly and properly filed and recorded the financing statement before the windows were installed. The owner stored the old windows in the basement of Blackacre. This year, the owner has suffered severe financial reverses and has defaulted on his mortgage obligation to the bank and on his obligation to the supplier of the windows. The bank brought an appropriate action to enjoin the supplier from its proposed repossession of the window inserts. In the action, the court should rule for the bank, because its mortgage was recorded first. the bank, because windows and screens, no matter their characteristics, are an integral part of a house. the supplier, because the inserts are removable. the supplier, because the availability of the old windows enables the bank to return Blackacre to its original condition.

Answer choice C is correct. U.C.C. Article 9 gives a holder of a security interest in fixtures the right to remove the fixtures after default, as long as the fixtures are removable. Because the facts indicate the windows are removable, the supplier is entitled to remove them. Answer choice A is incorrect because Article 9 gives priority to a purchase money security interest in removable fixtures even over a prior recorded mortgage, but only as to those fixtures. The supplier has a purchase money security interest in the windows because was given in connection with the acquisition of the windows. Answer choices B and D are incorrect because the key issue is whether the windows are removable, which is clearly specified in the facts.

A seller entered into a written contract to sell her house and six acres known as Meadowacre to a buyer for $75,000. Delivery of the deed and payment of the purchase price were to be made six months after the contract. The contract provided that Meadowacre was to be conveyed "subject to easements, covenants, and restrictions of record." The contract was not recorded. After the contract was signed but before the deed was delivered, the electric company decided to run a high-voltage power line in the area and required an easement through a portion of Meadowacre. The seller, by deed, granted an easement to the electric company in consideration of $5,000; the deed was duly recorded. The power line would be a series of towers with several high-voltage lines that would be clearly visible from the house on Meadowacre but would in no way interfere with the house. When the buyer caused the title to Meadowacre to be searched, the deed of easement to the electric company was found. The seller appeared at the time and place scheduled for the closing and proffered an appropriate deed to the buyer and demanded the purchase price. The buyer refused to accept the deed. In an appropriate action for specific performance against the buyer, the seller demanded $75,000. In this action, the seller should A. obtain an order for specific performance at a price of $75,000. B. obtain an order for specific performance at a price of $70,000. C. lose, because the buyer did not contract to take subject to the easement to the electric company. D. lose, because a high-voltage power line is a nuisance per se.

Answer choice C is correct. Under the doctrine of equitable conversion, although the seller retains legal title during the pendency of the land sale contract, equitable title (and risk of loss) passes to the buyer upon entering the contract. This enables the buyer to act as the owner of the property without having legal title. Consequently, in this case the seller did not have the authority to grant the easement. Answer choices A and B are incorrect because, as the seller did not have authority to grant the easement, she cannot now force the buyer to perform on the contract subject to that easement. Answer choice D is incorrect because whether or not the power lines are a nuisance does not determine the outcome of the seller's action.

An owner was the title holder of Blackacre, a single-family residence. Fifteen years ago, the owner conveyed a life estate in Blackacre to her sister. Fourteen years ago, the sister, who had taken possession of Blackacre, leased Blackacre to a tenant for a term of 15 years at the monthly rental of $500. Eleven years ago, the sister died intestate leaving her son as her sole heir. The tenant regularly paid rent to the sister and, after the sister's death, to the son until last month. The period in which to acquire title by adverse possession in the jurisdiction is 10 years. In an appropriate action, the tenant, the owner, and the son each asserted ownership of Blackacre. The court should hold that title in fee simple is in A. the owner, because the owner held a reversion and the sister has died. B. the son, because the sister asserted a claim adverse to the owner when the sister executed a lease to the tenant. C. the son, because the tenant's occupation was attributable to the son, and the sister died 11 years ago. D. the tenant, because of the tenant's physical occupancy and because the tenant's term ended with the sister's death.

Answer choice C is correct. Upon the sister's death, her son adversely possessed the property by exercising ownership rights over Blackacre by continuing the rental arrangement with the tenant. The tenant has superior rights to Blackacre over the original owner, because the tenant was in possession of Blackacre for the 10-year adverse possession without acknowledging the original owner's rights to Blackacre. Answer choice A is incorrect because, despite ownership of Blackacre returning to the original owner upon the death of the owner's sister (as the holder of the life estate), the son is the current owner of Blackacre due to his adverse possession of the property for the requisite statutory period (10 years). Answer choice B is incorrect because the sister's lease to the tenant was not adverse; it was within the scope of her life estate interest. Answer choice D is incorrect because the tenant's act of paying rent indicates his acknowledgment of the son's superior rights.

A landlord, the owner of the only shopping center in a small town, and a tenant, a new small-business owner, entered into a lease for a commercial shopping space in the shopping center. The tenant was unsure that the community would support her new business, so she wanted to limit the terms of the lease to a maximum of two years. The landlord, however, insisted on an at-will tenancy for a minimum of ten years, and included the following clause in the lease: "pursuant to this Lease, Landlord is given the express right to terminate the leasehold with Tenant by giving 30 days' notice." The lease omitted language giving the tenant a similar termination right. Due to the lack of commercial space available to rent in the area, the tenant agreed. Six months into the lease, the tenant terminated the lease in writing with 30 days' notice, explaining that, although sales at the shopping space were technically covering all of the tenant's expenses, the tenant had found lower rent in a nearby town, which she believed would be a more successful market. The landlord sued the tenant for breach of the lease. Will the landlord prevail in the breach of lease action? A. Yes, because the lease contract reserved the right of termination only for the landlord. B. Yes, because the tenant's reasons for terminating the lease were in bad faith. C. No, because termination rights in at-will tenancies cannot be limited. D. No, because the lease's unconscionability gave the tenant the right to terminate the lease.

Answer choice D is correct. A tenancy at will is a leasehold estate that may be terminated by either the landlord or the tenant. If only one party is expressly given the right to terminate the leasehold, the lease may, when taking into account all other circumstances, be unconscionable. In such a case, either party is given the ability to terminate the lease. In this case, the landlord—being the owner of the only commercial shopping center in town—had such superior bargaining power that enforcing the terms of the lease against the tenant would be unconscionable. Accordingly, both parties had the right to terminate this lease. Answer choice A is incorrect because an at-will tenancy terminable by only one party is often unconscionable, and if so, the tenant's right of termination exists by implication. Answer choice B is incorrect because good faith is irrelevant in determining whether a party validly terminated an at-will tenancy; here, the tenant gave proper notice of termination. Answer choice C is incorrect because termination rights in at-will tenancies can be limited; for example, a landlord is not granted an implied right of termination when a lease expressly gives a tenant the right of termination but is silent as to the landlord.

At a time when the owner held Lot 1 in the Fairoaks subdivision in fee simple, a man executed a warranty deed that recited that the man conveyed Lot 1 in Fairoaks to a buyer. The deed was promptly and duly recorded. After the recording of the deed from the man to the buyer, the owner conveyed Lot 1 to the man by a warranty deed that was promptly and duly recorded. Later, the man conveyed the property to a second buyer by warranty deed and the deed was promptly and duly recorded. The second buyer paid the fair market value of Lot 1 and had no knowledge of any claim of the first buyer. In an appropriate action, the second buyer and the first buyer contest title to Lot 1. In this action, judgment should be for the first buyer, because the first buyer's deed is senior to the second buyer's deed. the second buyer, because the second buyer paid value without notice of the first buyer claim. the first buyer or the second buyer, depending on whether a subsequent grantee is bound, at common law, by the doctrine of estoppel by deed. the first buyer or the second buyer, depending on whether the first buyer's deed is deemed recorded in the second buyer's chain of title.

Answer choice D is correct. Although an instrument is recorded, it may not be recorded in such a way as to give notice to subsequent purchasers (i.e., the deed may not be in the "chain of title"). A deed not within the chain of title is a "wild deed." Even though it is a senior deed, the deed from the man to the first buyer is wild; accordingly, answer choice A is incorrect. Answer choice B is incorrect because the facts do not state that this jurisdiction's recording statue is notice or race-notice. Answer choice C is incorrect because, although a grantor who conveys interest to land by warranty deed before actually owning it is estopped from later denying the effectiveness of her deed and the title is then transferred automatically to the prior grantee when the grantor acquires it, the question is whether a subsequent purchaser obtains good title from that same grantor. The majority rule is that a subsequent bona fide purchaser can obtain good title, despite the doctrine of estoppel by deed.

An owner was the title holder of several vacant lots in a subdivision. She obtained a $50,000 loan from a bank, and executed and delivered to the bank a promissory note and mortgage describing Lots 1, 2, 3, 4, and 5. The mortgage was promptly and properly recorded. Upon payment of $10,000, the owner obtained a release of Lot 2 duly executed by the bank. She altered the instrument of release to include Lot 5 as well as Lot 2 and recorded it. The owner thereafter sold Lot 5 to an innocent purchaser for value. The bank discovered that the instrument of release had been altered and brought an appropriate action against the owner and the purchaser to set aside the release as it applied to Lot 5. The owner did not defend against the action, but the purchaser did. The recording act of the jurisdiction provides: "No unrecorded conveyance or mortgage of real property shall be good against subsequent purchasers for value without notice, who shall first record." The court should rule for A. the purchaser, because the bank was negligent in failing to check the recordation of the release. B. the purchaser, because she was entitled to rely on the recorded release. C. the bank, because the purchaser could have discovered the alteration by reasonable inquiry. D. the bank, because the alteration of the release was ineffective.

Answer choice D is correct. Although purchasers who give value in good faith and without notice are generally protected, this is not the case when fraud is involved. Because the owner forged the release, it was ineffective, even against the purchaser (who was a bona fide purchaser). Thus, answer choice B is incorrect. Answer choice A is incorrect because the bank does not have any duty to check the recordation of the release. Answer choice C is incorrect because answer choice D is a better answer; the forged release is ineffective whether or not the purchaser knew or should have known that it was a forgery.

A rich man owned two adjacent parcels, Blackacre and Whiteacre. Blackacre fronts on a poor unpaved public road, while Whiteacre fronts on Route 20, a paved major highway. Fifteen years ago, the rich man conveyed to his son Blackacre "together with a right-of-way 25 feet wide over the east side of Whiteacre to Route 20." At that time, Blackacre was improved with a ten-unit motel. Ten years ago, the rich man died. His will devised Whiteacre "to my son, for life, remainder to my daughter." Five years ago, the son executed an instrument in the proper form of a deed, purporting to convey Blackacre and Whiteacre to a local businessman in fee simple. The businessman then enlarged the motel to 12 units. Six months ago, the son died and the daughter took possession of Whiteacre. She brought an appropriate action to enjoin the businessman from using the right-of-way. In this action, who should prevail? The daughter, because merger extinguished the easement. The daughter, because the businessman has overburdened the easement. The businessman, because he has an easement by necessity. The businessman, because he has the easement granted by the rich man to his son.

Answer choice D is correct. An easement is of infinite duration unless otherwise limited. The rich man granted his son an express easement on Whiteacre, which was never terminated, and which benefit followed Blackacre when the son conveyed it to the businessman. The burden of that easement followed Whiteacre, so the businessman has the easement over the daughter's property. Because the son owned a fee simple in Blackacre and a life estate in Whiteacre, he did not own the two estates equally at any one time, and thus they did not merge (making answer choice A is incorrect). Answer choice B is incorrect because nothing in the fact pattern indicates that an increase from 10 to 12 units would overburden the easement. Answer choice C is incorrect because the easement was expressly granted, and because an easement cannot be implied by necessity when an alternate route exists, as is the case here via the unpaved public road.

An owner owned in fee simple two adjoining lots, Lots 1 and 2. He conveyed in fee simple Lot 1 to a grantee. The deed was in usual form of a warranty deed with the following provision inserted in the appropriate place: "Grantor, for himself, his heirs and assigns, does covenant and agree that any reasonable expense incurred by grantee, his heirs and assigns, as the result of having to repair the retaining wall presently situated on Lot 1 at the common boundary with Lot 2, shall be reimbursed one-half the costs of repairs, and by this provision the parties intend a covenant running with the land." The grantee conveyed Lot 1 in fee simple to a woman by warranty deed in usual and regular form. The deed omitted any reference to the retaining wall or any covenant. Fifty years after the original owner's conveyance to the grantee, the woman conveyed Lot 1 in fee simple to a man by warranty deed in usual form; this deed omitted any reference to the retaining wall or the covenant. There is no statute that applies to any aspect of the problems presented except a recording act and a statute providing for acquisition of title after ten years of adverse possession. All conveyances by deed were for a consideration equal to fair market value. The deed from the original owner to the first grantee was never recorded. All other deeds were promptly and properly recorded. Lot 2 is now owned by the original owner's son, who took by intestate succession from the owner, now dead. The man expended $3,500 on the retaining wall. Then he obtained all of the original deeds in the chain from the original owner to him. Shortly thereafter, the man discovered the covenant in the original owner's deed to the first grantee. He demanded that the son pay $1,750, and when the son refused, the man instituted an appropriate action to recover that sum from the son. In such action, the son asserted all defenses available to him. If judgment is for the son, it will be because the man is barred by adverse possession. the first grantee's deed from the owner was never recorded. the man did not know about the covenant until after he had incurred the expenses and, hence, could not have relied on it. the man's expenditures were not proved to be reasonable and customary.

Answer choice D is correct. As the owner's successor in interest, the son took the owner's obligation under the covenant. The covenant requires the son to pay half of the man's reasonable expenses, so the only way the son could win is if the expenses incurred by the man were unreasonable. Answer choice A is incorrect because there was no possession in the fact pattern that is adverse. Answer choice B is incorrect because the son was not a subsequent purchaser of Lot 1 protected under the recording statute. Instead, he inherited title to Lot 2, along with the obligations under the covenant that ran with the land. Answer choice C is incorrect because reliance on the covenant is not required to enforce it.

The owner in fee simple of a small farm consisting of thirty acres of land improved with a house and several outbuildings leased the same to a tenant for a ten-year period. After two years had expired, the government condemned twenty acres of the property and allocated the compensation award to the owner and the tenant according to their respective interest so taken. It so happened, however, that the twenty acres taken embraced all of the farm's tillable land, leaving only the house, outbuildings, and a small woodlot. There is no applicable statute in the jurisdiction where the property is located, nor any provision in the lease relating to condemnation. The tenant quit possession, and the owner brought suit against him to recover rent. The owner will lose, because there has been a frustration of purpose which excuses the tenant from further performance of his contract to pay rent. lose, because there has been a breach of the implied covenant of quiet enjoyment by the owner's inability to provide the tenant with possession of the whole of the property for the entire term. win, because of the implied warranty on the part of the tenant to return the demised premises in the same condition at the end of the term as they were at the beginning. win, because the relationship of landlord and tenant was unaffected by the condemnation, thus leaving the tenant still obligated to pay rent.

Answer choice D is correct. Because a tenant must pay a reasonable rental value of the premises occupied if there has been a partial eviction by a third party with a superior claim to the property, the tenant must pay rent for the remaining property. Answer choice A is incorrect. Frustration of purpose occurs when the entire purpose of the contract has been destroyed. Because it is unclear if the tenant was farming on the land or contracted the land for some other purpose, answer choice A cannot be chosen. Answer choice B is incorrect because the owner is not responsible for the partial eviction. Answer choice C is incorrect because the majority view today is that the tenant is excused from paying rent if the premises are destroyed, as long as the tenant is not at fault for the destruction.

An owner owned Blackacre, her home. Her daughter lived with her and always referred to Blackacre as "my property." Two years ago, the daughter for a valuable consideration, executed and delivered to a buyer an instrument in the proper form of a warranty deed purporting to convey Blackacre to the buyer in fee simple, reserving to herself an estate for two years in Blackacre. The buyer promptly and properly recorded his deed. One year ago, the owner died and by will, duly admitted to probate, left her entire estate to the daughter. One month ago, the daughter, for a valuable consideration, executed and delivered to her friend an instrument in the proper form of a warranty deed purporting to convey Blackacre to the friend, who promptly and properly recorded the deed. The daughter was then in possession of Blackacre and the friend had no actual knowledge of the deed to the buyer. Immediately thereafter, the daughter gave possession to the friend. The recording act of the jurisdiction provides: "No conveyance or mortgage of real property shall be good against subsequent purchasers for value and without notice unless the same be recorded according to law." Last week, the daughter fled the jurisdiction. Upon learning the facts, the friend brought an appropriate action against the buyer to quiet title to Blackacre. If the friend wins, it will be because the daughter had nothing to convey to the buyer two years ago. the daughter's deed to the buyer was not to take effect until after the daughter's deed to the friend. the friend was first in possession. the daughter's deed to the buyer was not in the friend's chain of title.

Answer choice D is correct. Because only bona fide purchasers without notice of a prior claim prevail in notice jurisdictions, the friend can only win if he had no notice of the daughter-buyer deed. If a title search failed to show the deed (because no search of transactions by the owner would show title going to the buyer), the friend would have no notice of the "wild deed" because it was outside the friend's chain of title. Answer choice A is incorrect because, under the doctrine of estoppel by deed, the daughter would be estopped from denying the effectiveness of her deed to the buyer. Answer choice B is incorrect because the facts indicate the daughter's intention to transfer immediate title to the buyer upon delivery. Answer choice C is incorrect because possession is not a determining factor on these facts.

In 1990, the owner of a 300-acre tract, prepared and duly recorded a subdivision plan called Quiet Meadows. The plan showed 90 three-acre lots and a thirty-acre tract in the center that was designated "Future Public School." The owner published and distributed a brochure promoting Quiet Meadows which emphasized the proximity of the lots to the school properly and indicated potential tax savings "because the school district will not have to expend tax money to acquire this property." There is no specific statute concerning the dedication of school sites. The owner sold 50 of the lots to individual purchasers. Each deed referred to the recorded plan. In 1996, the owner sold the remaining 40 lots and the thirty-acre tract to a buyer by a deed that referred to the plan. The buyer sold the 40 lots to individual purchasers and the thirty-acre tract to a restaurateur. None of the deeds from the buyer referred to the plan. The school board of the district in which Quiet Meadows is situated has voted to erect a new school on the thirty-acre tract. In an appropriate action between the school board and the restaurateur to determine title, the result will be in favor of A. the restaurateur, because the school board has been guilty of laches. B. the restaurateur, because his deed did not refer to the subdivision plan. C. the school board, because the restaurateur had constructive notice of the proposed use of the tract. D. the school board, because there has been a dedication and acceptance of the tract.

Answer choice D is correct. Because there is no statute governing dedication for schools, the owner's deed is sufficient to establish a dedication of land for public use. Answer choice A is incorrect because there was no unreasonable delay in asserting their rights. Answer choice B is incorrect because the prior deed referred to the recorded plan containing the dedication. Answer choice C is incorrect because, even though the restaurateur had notice of the proposed use, this answer choice does not establish why the school board is the rightful owner. The foregoing NCBE MBE question has been modified to reflect current NCBE stylistic approaches; the NCBE has not reviewed or endorsed this modification.

A man and woman were neighbors whose small yards were separated only by small bushes. After a discussion about building a one-foot thick stone wall to separate the two properties, the neighbors agreed that the man would pay for the wall, as the woman did not have the funds to do so, and as a consolation, the wall would be built on the woman's property so as to not reduce the square footage of the man's yard. Years later, the woman sold her property in a valid transaction with a buyer, but she told the buyer that the man had actually paid for and built the wall and that she had agreed to keep it there. Regardless, the buyer then spoke to the man about her desire to tear down the wall to open up the space and stated that she would pay for the destruction of the wall. The man objected to tearing down the wall. May the man prevent the buyer from tearing down the wall? A. Yes, because he has a separate security interest in the materials used to build the wall. B. Yes, because he paid for materials and construction of the wall. C. No, because the wall constituted an easement in gross. D. No, because the buyer validly purchased the land from the woman.

Answer choice D is correct. Fixtures, or structures built on real property, become part of the realty. Therefore, the wall became part of the woman's land, which was then sold to the buyer. When the buyer bought the woman's property, the wall, as a fixture, was included in the sale—whether the man paid for its construction or not. Absent some other legal prohibition not mentioned in the facts, the buyer could unilaterally decide to tear down the wall. Answer choice A is incorrect because once the materials were incorporated into the wall, they became an integral part of the property, and the man was not subject to any security interest in the materials. Answer choice B is incorrect because although the man paid for the wall, he had no security interest and no right to object to the wall's destruction. Answer choice C is incorrect because the wall did not constitute an easement on the woman's property. While easements in gross are not tied to the land, that is, they remain tied to one person, the relevant person is the owner of the dominant estate (here, the man), not the owner of the servient estate (here, the woman). Therefore, even if the wall did constitute an easement, it would benefit the man, and the buyer would be prevented from tearing it down. However, the wall would be more accurately characterized as a fixture, and therefore the buyer is the new owner of the wall.

A woman died testate. In her will, she devised a farm she owned to her husband for life, remainder to her niece. Her will did not specify the duties of the husband and the niece with regard to maintenance and expenses related to the farm. The husband took sole possession of the farm, did not farm the land, and did not rent the land to a third person, although the fair rental value was substantial. For two years in a row after the woman died, the county assessor sent the tax bills to the niece, but the niece did not pay the bills, because she and the husband could not agree on who should pay them. Finally, the niece paid the taxes to avoid a tax foreclosure sale. The niece then sued the husband for reimbursement for the two years' worth of property taxes. There is no applicable statute. Is the niece likely to prevail? A. No, because remaindermen are solely responsible for the payment of property taxes. B. No, because the county assessor sent the bills to the niece. C. No, because the woman's will was silent on responsibility for payment of property taxes. D. Yes, because the niece paid an obligation that was the sole responsibility of the husband.

Answer choice D is correct. In the absence of a contrary direction in the document creating the life estate - in this case, the will - it is the duty of the life tenant to pay all general property taxes that accrue during the continuance of the life estate. The only limitation on this duty is that the life tenant has no duty to expend more than the income that can be generated from the land. Because the fair rental value of the farmland was substantial, this limitation does not apply. If the remainderman does pay any property taxes due during the life tenancy, he or she is entitled to a judgment against the life tenant for reimbursement. Answer choice A is incorrect for the reasons stated above. Answer choice B is incorrect because it is not relevant to whom the county assessor sent the tax bills. The husband is responsible for the property taxes because the fair rental value of the farmland was substantial. As previously explained, answer choice C is incorrect.

A corporation owned Blackacre in fee simple, as the real estate records showed. The corporation entered into a valid written contract to convey Blackacre to a buyer, an individual. At closing, the buyer paid the price in full and received an instrument in the proper form of a deed, signed by duly authorized corporate officers on behalf of the corporation, purporting to convey Blackacre to the buyer. The buyer did not then record the deed or take possession of Blackacre. Next, a creditor (who had no knowledge of the contract or the deed) obtained a substantial money judgment against the corporation. Then, the buyer recorded the deed from the corporation. Thereafter, the creditor properly filed the judgment against the corporation. A statute of the jurisdiction provides: "Any judgment properly filed shall, for ten years from filing, be a lien on the real property then owned or subsequently acquired by any person against whom the judgment is rendered." Afterward, the buyer entered into a valid written contract to convey Blackacre to a purchaser. The purchaser objected to the buyer's title and refused to close. The recording act of the jurisdiction provides: "Unless the same be recorded according to law, no conveyance or mortgage of real property shall be good against subsequent purchasers for value and without notice." The buyer brought an appropriate action to require the purchaser to complete the purchase contract. The court should decide for the purchaser, because the creditor's judgment was obtained before the buyer recorded the deed from the corporation. the purchaser, because even though the corporation's deed to the buyer prevented the creditor's judgment from being a lien on Blackacre, the creditor's filed judgment poses a threat of litigation. the buyer, because the buyer recorded her deed before the creditor filed his judgment. the buyer, because the buyer received the deed from the corporation before the creditor filed his judgment.

Answer choice D is correct. The statute clearly restricts the judgment lien to property then or subsequently owned, not property owned prior to the judgment. The buyer had title before the creditor's judgment was filed, which means the judgment cannot apply to her property. The references throughout the question and answer choices to the buyer's failure to record the deed are confusing and distract from the analysis of the question, which has nothing to do with the recording statute. Because the corporation did not own Blackacre at the time of the judgment, the judgment did not create a lien on the title, the buyer's title is unencumbered, and there is no threat of litigation. Answer choice A is incorrect because, as explained above, the recording is irrelevant, as the judgment was not obtained until after the deed had been transferred. Answer choice B is incorrect because title to Blackacre is marketable. Answer choice C is incorrect because the time of recording is not at issue; it is the time of transfer of the deed that is relevant, as explained above.


Kaugnay na mga set ng pag-aaral

Which Resources are found in Florida?

View Set

Social Media Marketing Final (Review)

View Set

MedSurg Comprehensive Final Examination

View Set

Infant and Childhood Development: Final Exam

View Set

AAAAAAAAAAA, EMT Chap 40, and 41 quiz review, chapter 37-40 test exam 2, Chapter 37-40

View Set

Among the Hidden Vocab Chapters 12-15

View Set